Question 15.CA.7: Determine the reactions at the supports for the prismatic be...

Determine the reactions at the supports for the prismatic beam and loading shown in Fig. 15.19a. (This is the same beam and loading as in Concept Application 15.5.)

15.7
The blue check mark means that this solution has been answered and checked by an expert. This guarantees that the final answer is accurate.
Learn more on how we answer questions.

We consider the reaction at B as redundant and release the beam from the support. The reaction \pmb{R}_B is now considered as an unknown load (Fig. 15.19b) and will be determined from the condition that the deflection of the beam at B must be zero. The solution is carried out by considering separately the deflection (y_B)_w caused at B by the uniformly distributed load w (Fig. 15.19c) and the deflection (y_B)_R produced at the same point by the redundant reaction \pmb{R}_B (Fig. 15.19d).

From the table of Appendix E (cases 2 and 1),

Beam and Loading Elastic Curve Maximum Deflection Slop at End Equation of Elastic Curve
-\frac{P L^3}{3 E I} -\frac{P L^2}{2 E I} y=\frac{P}{6 E I}\left(x^3-3 L x^2\right)
-\frac{w L^4}{8 E I} -\frac{w L^3}{6 E I} y=-\frac{w}{24 E I}\left(x^4-4 L x^3+6 L^2 x^2\right)
-\frac{M L^2}{2 E I} -\frac{M L}{E I} y=-\frac{M}{2 E I} x^2
-\frac{P L^3}{48 E I} \pm \frac{P L^2}{16 E I} For x \leq \frac{1}{2} L:
y=\frac{P}{48 E I}\left(4 x^3-3 L^2 x\right)
For a > b:
-\frac{P b\left(L^2-b^2\right)^{3 / 2}}{9 \sqrt{3} E I L}
at x_m=\sqrt{\frac{L^2-b^2}{3}}
\theta_A=-\frac{P b\left(L^2-b^2\right)}{6 E I L}
\theta_B=+\frac{P a\left(L^2-a^2\right)}{6 E I L}
For x < a:
y=\frac{P b}{6 E I L}\left[x^3-\left(L^2-b^2\right) x\right]
For x = a:    y=-\frac{P a^2 b^2}{3 E I L}
-\frac{5 w L^4}{384 E I} \pm \frac{w L^3}{24 E I} y=-\frac{w}{24 E I}\left(x^4-2 L x^3+L^3 x\right)
\frac{M L^2}{9 \sqrt{3} E I} \theta_A=+\frac{M L}{6 E I}
\theta_B=-\frac{M L}{3 E I}
y=-\frac{M}{6 E I L}\left(x^3-L^2 x\right)
\left(y_B\right)_w=-\frac{w L^4}{8 E I} \quad\left(y_B\right)_R=+\frac{R_B L^3}{3 E I}

Writing that the deflection at B is the sum of these two quantities and that it must be zero,

\begin{aligned}&y_B=\left(y_B\right)_w+\left(y_B\right)_R=0 \\&y_B=-\frac{w L^4}{8 E I}+\frac{R_B L^3}{3 E I}=0\end{aligned}

and, solving for R_B, \quad R_B=\frac{3}{8} w L \quad \pmb{R}_B=\frac{3}{8} w L \uparrow

Drawing the free-body diagram of the beam (Fig. 15.19e) and writing the corresponding equilibrium equations,

\begin{aligned}+\uparrow \Sigma F_y=0: \quad &R_A+R_B-w L=0 \\&R_A=w L-R_B=w L-\frac{3}{8} w L=\frac{5}{8} w L \\&\pmb{R}_A=\frac{5}{8} w L \uparrow\end{aligned}     (1)

\begin{aligned}+\circlearrowleft \Sigma M_A=0 \quad M_A+R_B L-(w L)\left(\frac{1}{2} L\right)&=0 \\M_A=\frac{1}{2} w L^2-R_B L=\frac{1}{2} w L^2-\frac{3}{8} w L^2&=\frac{1}{8} w L^2 \\\pmb{M}_A&=\frac{1}{8} w L^2 \circlearrowleft\end{aligned}     (2)

Alternative Solution. We may consider the couple exerted at the fixed end A as redundant and replace the fixed end by a pin-and-bracket support. The couple \pmb{M}_A is now considered as an unknown load (Fig. 15.19f) and will be determined from the condition that the slope of the beam at A must be zero. The solution is carried out by considering separately the slope (θ_A)_w caused at A by the uniformly distributed load w (Fig. 15.19g) and the slope (θ_A)_M produced at the same point by the unknown couple \pmb{M}_A (Fig. 15.19h).

Using the table of Appendix E (cases 6 and 7) and noting that A and B must be interchanged in case 7,

\left(\theta_A\right)_w=-\frac{w L^3}{24 E I} \quad\left(\theta_A\right)_M=\frac{M_A L}{3 E I}

Writing that the slope at A is the sum of these two quantities and that it must be zero gives

\begin{aligned}&\theta_A=\left(\theta_A\right)_w+\left(\theta_A\right)_M=0 \\&\theta_A=-\frac{w L^3}{25 E I}+\frac{M_A L}{3 E I}=0\end{aligned}

where M_A is

M_A=\frac{1}{8} w L^2 \quad \pmb{M}_A=\frac{1}{8} w L^2 \circlearrowleft

The values of R_A and R_B are found by using the equilibrium equations (1) and (2).

15.7-
15.7--
15.7---

Related Answered Questions

Question: 15.CA.5

Verified Answer:

Equilibrium Equations. From the free-body diagram ...
Question: 15.CA.3

Verified Answer:

Divide the beam into two portions, AD and DB, and ...